AAMC CBT7 and 7R OFFICIAL Q&A

This forum made possible through the generous support of SDN members, donors, and sponsors. Thank you.

Vihsadas

No summer
Moderator Emeritus
Lifetime Donor
10+ Year Member
5+ Year Member
15+ Year Member
Joined
Oct 17, 2007
Messages
5,474
Reaction score
56
This is the official Q&A thread for AAMC CBT7 and 7R.

Please post ONLY questions pertaining to AAMC CBT7 and 7R.
Out of respect for people who may not have completed the other exams, do not post questions or material from any other AAMC exam.

Please see this thread for the rules of order before you post.

Good luck on your MCAT!

Members don't see this ad.
 
PS #52.

Question asking about the pH when the color change occurs depends on?

The answer is the pKa of the indicator. I know that the color changes at the equiv point, which depends on the initial concentration of acid - right?

Is it always true that when the indicator changes color (the equivalence point) the pH = pKa (of the indicator) ??

BS #100

since when is an alcohol more polar than a ketone. The carbonyl carbon has a partial (+) and oxygen has a patrial (-) thats why its so reactive to nucleopihilic attack. Anyone have a source that talks about he polarity of alcohols vs ketones??


Earthquake passage was total crap :p
 
The alcohol is more polar is because of the hydrogen bonding.

Remember if there is no indicated charge on the compound, then you refer to hydrogen boding for polarity. Because the alcohol has its Oxygen hydrogen bonded to hydrogen, it is more molar than the regular ketone.
 
BS #100

since when is an alcohol more polar than a ketone. The carbonyl carbon has a partial (+) and oxygen has a patrial (-) thats why its so reactive to nucleopihilic attack. Anyone have a source that talks about he polarity of alcohols vs ketones??

Think of the difference in electronegativity for an oxygen bonded to carbon, and an oxygen bonded to hydrogen. The electronegativity will be greater for an oxygen bonded to hydrogen, which creates a stronger dipole moment; thus, making the molecule more polar than a ketone.
 
Members don't see this ad :)
Verbal was insane in this test..got a 9 so I'm happy.

PS was just...ugh. It wasn't HARD but I just didn't enjoy it much at all. Got a 10 which is my lowest...I failed all the optics stand alones so thats a big reason I didn't get a 11.

Also ran out of time and had to literally just circle 3 on three questions...
 
For number 31, the aamc explanation for their answer was based on the use of the eqn Intensity=power/area=P/r^2.

All of their answers, however, were calculated without squaring the mileage (the r) in the denominator.

Can someone explain this question and answer to me?
 
I wanted to ask about Q44 on 7CBT.

Question asks:

Because earthquakes were triggered by what ultimately must have been comparitively minor energy transfers from the distant Landers quake, the triggered quakes probably occurred where:

A) significant local stress forces the earth's crust were already in precarious equilbrium

B) the earth's crust was subjected to reinforcing resonant effects that cumulatively built up local stress


I left out C & D because they were absolutely wrong.

Now I was wondering what made A a better answer than B. Even the answer admits that B sounds like a good answer.

My problem with A being the answer is that the passage states that at a distance 4L the deformations are less than the daily distortions due to tidal forces.

Now, stresses are never mentioned, but I was thinking that if at 4L away the forces are already small enough such that they deform the earth as much as normal forces do. Yet triggered quakes were reported to occur up to 17L away. That would mean the stress/forces in question were tiny??

Does anyone have a definitive answer to this question - and any tips on how to handle these sort of "nonphysics" questions on the actual test??

THANKS!

I also missed this question....I retook the test...missed it both time with different answers. I really don't understand this, and I'm frustrated.

Any help would be greatly appreciated.
 
#138:

If the signaling interaction at the two-cell stage involves protein, then how could the actinomycin-treated embryo develop normally? If mRNA synthesis is inhibited then where exactly will the required protein come from? The oleosome ;)?

Somone enlighten me!
 
When looking at the phrase "% of dividing nuclei with radioactive label", you should be focusing more on "with radioactive label" than "dividing." I see where you're coming from, the word "dividing" is misleading since it makes you think of mitosis and should probably have been left out altogether.

Remember the cells are synchronized as stated in the question. If they all underwent mitosis during the time frame you were talking about, the PERCENTAGE of radioactive nuclei would not change. If before mitosis there were x% radioactive and y% non-radioactive then after mitosis there will be twice as many radioactive and twice as many non radioactive as before, but those x and y percentages would still be the same. Since there is no DNA synthesis going on nuclei that were non-radioactive before cannot suddenly become radioactive during mitosis.

DNA synthesis is the better answer it CAN account for a percentage increase in the number of radioactive nuclei, since DNA synthesis converts formerly non-radioactive nuclei into radioactive nuclei, thus increasing the percentage of radioactive ones.


But what explains the decline?
 
VR, #69: I chose A but they say it's wrong.

"Which of the following ideas about humans is clearly NOT assumed in the passage?

A) Humans lack the sensory means to detect some intraspecies messages."

Explanation:

"The author strongly suggests that humans possess these sensory means through direct observation, especially in the passage about the bamboo blooming once a century."

If the author suggests that humans "possess these sensory means", that means that they don't lack them. If they don't lack them then it is clearly NOT assumed that they do lack them.

Yea, I know; I'm also dizzy.
 
Does anyone else feel that sometimes AAMC verbal passages are poorly written? I got a 10 on this verbal but I thought the first passage on biodiversity was VERY poorly written. The author starts off by talking about biodiversity/extinction and then starts ranting about human uniqueness. I got every question right on this passage, but when I was reviewing it, I kept thinking to myself if this was a student and I was professor grading a paper in college, I would've given this paper a D or something. This is the most poorly written passage I've ever come across in my VR practice!
 
Question 20 in PS:

A gas that that occupies 10 L at 1 atm and 25 degrees celsius will occupy what volume at 500 atm and 25 degrees celsius?

Answer: somewhat more than 0.02 L because of the space occupied by the individual gas molecules. The ideal gas law makes the assumption that molecules have no volume. This assumption is adequate when the gas is at 1 atm but when the pressure is increased to 500 atm the volume of the gas molecules is no longer negligible.

Isn't the volume no longer negligible when the volume INCREASES? That is, if the pressure were to decrease and the volume were to increase?
 
Question 20 in PS:

A gas that that occupies 10 L at 1 atm and 25 degrees celsius will occupy what volume at 500 atm and 25 degrees celsius?

Answer: somewhat more than 0.02 L because of the space occupied by the individual gas molecules. The ideal gas law makes the assumption that molecules have no volume. This assumption is adequate when the gas is at 1 atm but when the pressure is increased to 500 atm the volume of the gas molecules is no longer negligible.

Isn't the volume no longer negligible when the volume INCREASES? That is, if the pressure were to decrease and the volume were to increase?

The volume is decreasing since the pressure is increasing. the answer explanation is clear. The significance of molecule's volume starts to take hold. Thus, the actual volume willbe greater than 0.02 L.
 
The volume is decreasing since the pressure is increasing. the answer explanation is clear. The significance of molecule's volume starts to take hold. Thus, the actual volume willbe greater than 0.02 L.

Yes, I can see that, as that's what the explanation says. That doesn't help me, though.

Anyone else?
 
Members don't see this ad :)
PS #52.

Question asking about the pH when the color change occurs depends on?

The answer is the pKa of the indicator. I know that the color changes at the equiv point, which depends on the initial concentration of acid - right?

Is it always true that when the indicator changes color (the equivalence point) the pH = pKa (of the indicator) ??

----------------->

I don't get it either... :( Someone please help.

----------->

I thought the verbal was insane too, most boring passages ever, this was my first cbt aamc exam, sorta disappointed.
 
Just took CBT7. Will be reviewing it tonight so I will be reading all your posted questions and answers. Did horrible once again on the VR. So far, VR 6&7 have been my lowest with an average of 7. BTW, the earthquake passage in PS was rough. I haven't looked back at it but I did a whole lot of guessing. But seeing as I pulled a 10 on PS, I prob. guessed right? Or completely missed the passage and destroyed the rest. I'm guessing both :oops:. 4 MORE DAYS!!
 
But what explains the decline?

I was also wondering about this. The steep incline makes sense since nuclei are incorporating the radioactive nucleotide into the cell, but the decline of radioactivity after this point doesn't make sense to me.
 
PS #52.

Question asking about the pH when the color change occurs depends on?

The answer is the pKa of the indicator. I know that the color changes at the equiv point, which depends on the initial concentration of acid - right?

Is it always true that when the indicator changes color (the equivalence point) the pH = pKa (of the indicator) ??

----------------->

I don't get it either... :( Someone please help.

----------->

I thought the verbal was insane too, most boring passages ever, this was my first cbt aamc exam, sorta disappointed.

An indicator is usually a weak acid, which dissociates and produces the color change. The dissociation of a weak acid depends upon its pKa.
 
Just wanted to add to that explanation about indicators.

The indicator changes color when the pH of the solution is equal to the pKa of the indicator - this always holds true. To see why, look at the Henderson-Hasselbach equation: pH = pKa + log([base]/[acid]). An indicator changes color when it gains or loses a proton, since the indicator itself is a weak acid. When the concentrations of the indicator in it's two different coloured forms (base and acid) are equal, the colour change starts being observed. According to the Henderson-Hasselbach equation, when [base] = [acid], log([base]/[acid]) = log(1) = 0 and so pH = pKa at the colour change. Therefore, the indicator always undergoes a colour change when the pH of the solution is equal to the indicator's pKa.

So, to choose a good indicator for a titration, you'd want an indicator whose pKa matches as closely as possible with the pH of the equivalence point of the titration. For example, in a strong acid/strong base titration, the equivalence pH is 7, so you'd want to choose an indicator with a pKa of approximately 7 in order to see the colour change at the appropriate time.

Hope that helped!
 
Can anyone clarify #8 from PS???

They seemed to have used the simple harmonic movement analogy but if one were to think along the electrostatic analogy, then you would use the following formula for electrical potential energy: EP= KQq/r and since at max displacement, r is at a max value, then EP should be the smallest shouldn't it?????

In Figure 1, the maximum electrical potential energy occurs at:
A) A only.
B) B only.
C) C only.
D) A and C only.

The potential energy of an oscillatory motion is ½kx2 where x is the displacement. Since the maximum displacement occurs at A and C, answer D is justified.
 
Can anyone clarify #8 from PS???

They seemed to have used the simple harmonic movement analogy but if one were to think along the electrostatic analogy, then you would use the following formula for electrical potential energy: EP= KQq/r and since at max displacement, r is at a max value, then EP should be the smallest shouldn't it?????

In Figure 1, the maximum electrical potential energy occurs at:
A) A only.
B) B only.
C) C only.
D) A and C only.

The potential energy of an oscillatory motion is ½kx2 where x is the displacement. Since the maximum displacement occurs at A and C, answer D is justified.

The answer works if you consider EP as well. Since q and Q have opposite signs, then the EP is always a negative number (except at infinite distance, where EP=0). At a small r, like at B, the EP is at a minimum value since this is where it's most negative. As r gets larger, like at A and C, the EP becomes less negative, and therefore, these are the points where the EP is at its maximum value.

Hope that helped.
 
The answer works if you consider EP as well. Since q and Q have opposite signs, then the EP is always a negative number (except at infinite distance, where EP=0). At a small r, like at B, the EP is at a minimum value since this is where it's most negative. As r gets larger, like at A and C, the EP becomes less negative, and therefore, these are the points where the EP is at its maximum value.

Hope that helped.

That helped ALOT!!! I can't believe I didn't take the negative sign into account... if r increases, the ABSOLUTE value decreases, but since it is negative, this means it INCREASES!!!!

Much thanks!!!!

Mel:luck:
 
Untitled%20picture2.png
Untitled%20picture2.png
Hey Guys, the last # of the BS section (#146) really confused me... the thing is, in the chart they are measuring the amount of radioactive nuclei (basically the percentage of cells whose nuclei have radioactive thymine) from the sample of cells which they extract. The reason why this confuses me, is since DNA is replicated in a semi-conservative model, and since it had incorporated radioactive thymine, then the amount of radioactive nuclei should remain constant and not decrease... I have a feeling that in this question, they should rather imply "the relative amount of radioactivity within cell nuclei" and then, it will be obvious that during DNA synthesis, the amount will be larger compared to when the cell has separated its DNA, thus separating the amount of radioactivity within it.


In an experiment on the phases of the cell cycle, cultures of actively dividing, synchronized cells were exposed to radioactively labeled 2-deoxythymidine for 30 minutes, then rinsed to remove the unabsorbed label. At various times thereafter, groups of cells were removed from the cultures and the nuclei examined to determine their content of radioactive material. Results are shown in the figure below.
itdmedia.aspx

Based on the figure, what process was occurring during hours 3-13 after treatment with radioactive 2-deoxythymidine? A
) Mitosis B
) Meiosis C
) DNA synthesis
This item includes a description of an experiment in which actively-dividing, synchronized cells were exposed to radioactively labeled 2-deoxythymidine (the nitrogen base incorporated into DNA, but not RNA). After 30 minutes of exposure to the radioactively labeled substance, the cells were rinsed to remove unabsorbed label. At various times thereafter, a group of cells from the culture were examined to determine the quantity of radioactive material in the nuclei. The figure shows a peak in radioactivity between 3 and 13 hours after treatment. This peak represents DNA synthesis, since 2-deoxythymidine is a component of DNA, but not RNA or protein.

D
) RNA synthesis
 
To be most effective, a gene therapy for gastric cancer should be directed against:
Athe stomach epithelial cells that give rise to tumors. -- correct answer
explanation given by aamc:

According to the passage, infection by H. pylori increases one's risk of gastric cancer, but less than 25% of people affected ultimately develop such cancer. To be most effective, a gene therapy for gastric cancer should be directed against epithelial cells that give rise to tumors. The infection originates in the epithelial tissue, which is the same location of the cancers—and the first place to focus preventative measures.


B the antibody producing cells of the immune system.
C all the cells in the host's body.
D all known genes in the H. pylori genome.

my question:
In this question, why is d not an option? I didn't choose a because I thought targeting the GI epithelial cells could mean disrupting GI function, which is problematic
 
To be most effective, a gene therapy for gastric cancer should be directed against:
Athe stomach epithelial cells that give rise to tumors. -- correct answer
explanation given by aamc:

According to the passage, infection by H. pylori increases one’s risk of gastric cancer, but less than 25% of people affected ultimately develop such cancer. To be most effective, a gene therapy for gastric cancer should be directed against epithelial cells that give rise to tumors. The infection originates in the epithelial tissue, which is the same location of the cancers—and the first place to focus preventative measures.


B the antibody producing cells of the immune system.
C all the cells in the host’s body.
D all known genes in the H. pylori genome.

my question:
In this question, why is d not an option? I didn't choose a because I thought targeting the GI epithelial cells could mean disrupting GI function, which is problematic

Its the human's epithelial cells that are getting cancer so you want to treat those not the bacteria. If anything you want to eliminate the bacterias not incorporate genes into them.
 
Its the human's epithelial cells that are getting cancer so you want to treat those not the bacteria. If anything you want to eliminate the bacterias not incorporate genes into them.

does gene therapy always imply "incorporating" genes?
 
well, now that i think about it, what sort of genes would you incorporate into human stomach epi cells to stop the cancer? genes that code for apoptosis inducing factor or something?
 
can someone please explain why tertiary alcohol is more reactive with HCL than the others ?

thanks
test-7-101.jpg
 
Chlorination of an alcohol usually results by the loss of the H2O (after OH's protonation by the alcohol)

therefore it is usually an SN1 reaction. And which molecule of those three is best suited for SN1? tert-b
 
EDIT: because antibodies and antibiotics are different things. This is why reviewing practice tests when half-falling-asleep doesn't do a whole lot of good.
 
Last edited:
Chlorination of an alcohol usually results by the loss of the H2O (after OH's protonation by the alcohol)

therefore it is usually an SN1 reaction. And which molecule of those three is best suited for SN1? tert-b

Thanks for the reply

but isn't SN2 rxns also result in loss of water ? (I just checked with TPR)
how come its SN1 and not SN2 ?
 
Thanks Doodl3s & dragon529 for your posts

However, I realize that it is easy to explain how the reaction takes place once you see what the right answer is (maybe I should have posted the image without the answer) I want to be able to solve this problem from scratch. ie. if the answer was not shown above how would you go about solving this problem ?

thanks for all your help... I really appreciate it


====================================
more specifically

Chlorination of an alcohol usually results by the loss of the H2O (after OH's protonation by the alcohol)

therefore it is usually an SN1 reaction. And which molecule of those three is best suited for SN1? tert-b

Both SN1 & SN2 involve the protonation of OH. How do I know if it will be a one step or two step mechanism if the answer above was not shown ?

thanks again
 
Last edited:
i don't think i've ever seen protonation of an alcohol followed by an SN2 to be honest... whenever i see protonation of an OH and subsequent loss of H2O, its always E1/SN1 in my mind... =/
 
what about number 20? i thought that Vreal<Videal and so i didnt choose any of the other options, atleast thats what TPR says. I know about the volume not being negligible in high pressure scenarios. Can someone clear this up for me?

thanks
 
i dont remember the question, but Real V is always GREATER than Ideal V...

and Real P is always LESS than Ideal P...

if thats what TPR says (Vreal<Videal) then TPR is WRONG, which is weird lol

Because ideal doesnt take into account molecular volume...
and ideal doesnt take into account intermolecular forces which decrease P.
 
i dont remember the question, but Real V is always GREATER than Ideal V...

and Real P is always LESS than Ideal P...

if thats what TPR says (Vreal<Videal) then TPR is WRONG, which is weird lol

Because ideal doesnt take into account molecular volume...
and ideal doesnt take into account intermolecular forces which decrease P.


that seems to make more sense! thanks! i appreciate it! stupid tpr.
 
i don't think i've ever seen protonation of an alcohol followed by an SN2 to be honest... whenever i see protonation of an OH and subsequent loss of H2O, its always E1/SN1 in my mind... =/

You're right, it never happens. SN2 involves a transition state, SN1 involves an intermediate. Protonating a hydroxyl group in order to make it a better LG implies that there is an intermediate involved.
 
sorry if this question has been answered already but i didn't see it so..

PS 27) Though 395-nm light is in the visible region of the electromagnetic spectrum, it is very near:
A. the radio wave region
B. the microwave region
C. the infrared region
D. the ultraviolet region <- Correct

Now if I had memorized the wavelengths of visible light these should have been super easy points. However, I didn't - so I went with info presented in the passage:

"Bromine is red-brown liquid that absorbs light very strongly at a wavelength of 395nm"

Using this info, I deduced that red light is at 395nm (incorrect), and therefore it is near the infrared region. I understand that this is a simple matter of memorization, but what is wrong with my reasoning as per info in the passage? Should I disregard passage info next time in favor of textbook info I know is correct?

Thanks.

Well, I don't think you should discard passage information. From what I can tell, things are in the passage for a reason, and if the passage gives numbers and equations, get ready to use them.

In your case, I think you might have read the information incorrectly. It was stated that Bromine absorbs light very strongly at 395nm, so it seems to me that it would show up with a strong, red color. It is red because the light you see is reflected.

Reflected color: Reflected color is color you can't see in the dark without another source of light (for example, a red apple). So, if a substance absorbs near 400, then it's color is the opposite (near 700 nm, or simply red). Do a search on google for the color wheel, and you might get an understanding of what I mean. So if something absorbs around the 700nm range, then it will probably appear green/blue (near the 400 nm range).

Emitted Color: Emitted color is color you can see in the dark, because it is emitting color and light. An example is a red firecracker. It appears red because it emitts red light (700nm) and this light that hits your eyes is not reflecting of anything. It is emitted at 700 nm and you see it as 700 nm


It's best to memorize the range
UV...400nm (blue)---------700nm (red)....IR


And know that the 400 nm is lower in wavelength, but higher in frequency and that the 700 nm is higher in wavelength, but lower in frequency. So the higher the frequency, the more damaging the light source (that's why too much UV radiation is not good for you).

Hope this helps pal!!
 
Got a 12/8/10

Did anyone think the VR was extremely difficult? I was guessing on so many questions.
 
I don't understand why C is not a plausible answer. I was down to B and C, and both seemed right to me.
 
Top